ارسال ها
10
لایک ها
3
امتیاز
0
پاسخ : ماراتن هندسه

در واقع باید ثابت کنیم CP زیر میانه ی مثلث ABC است. KL و MN را ادامه می دهیم تا یکدیگر را در D قطع کنند.خط CD ضلع AB را در T قطع میکند.CTt در واقع همان زیرنیانه است. زیرا:

پس کافیست ثابت کنیم خطوط CP و CT بر یکدیگر منطبق اند.
به وضوح KXND محاطی است و همچنین YMLD . ا P نیز بر این دو دایره قرار دارد. فرض کنید خط CD دایره ی محیطی KXN را در P' قطع کند.

پس P' بر دایره ی محیطی YMCD نیز قرار دارد.
پس P' همان P است.
پس CP همان CT است.

---- دو نوشته به هم متصل شده است ----

اگه درسته اینم سوال بعد: چهارضلعی ABCD مفروض است. نقاط X,Y,Z,T به ترتیب بر نیم خطهای BA,CB,CD,DA قرار دارند به طوریکه:

ثابت کنید X,Y,Z,T بر یک دایره یا بر یک خط اند. درضمن ثابت کنید اگر چهارضلعی ABCD محاطی باشد این چهار نقطه بر یک خطند.
 
آخرین ویرایش توسط مدیر
ارسال ها
10
لایک ها
3
امتیاز
0
پاسخ : ماراتن هندسه

فرض کنید این نقاط روی اضلاع چهارضلعی نیستند.
 
ارسال ها
10
لایک ها
3
امتیاز
0
پاسخ : ماراتن هندسه

قسمت دوم سوال کلا جداست. یعنی شرط لازم نیست. تو راه حل اصلی نمیشه حالات خط شدن رو شناسایی کرد.
راهنمایی: برای اثبات حالت کلی نقطه میشل چهار ضلعی را در نظر بگیرید.
و در حالتی که چهارضلعی محاطیه، نقاط داده شده بر روی خط هایی که از محل برخورد قطر ها موازی با اضلاع رسم میشه، قرار دارند.
 
ارسال ها
10
لایک ها
3
امتیاز
0
پاسخ : ماراتن هندسه

لم 1:در مثلث ABC، مماس وارد بر دایره محیطی در نقطه A، امتداد ضلع BC را در D قطع می کند.حال داریم:

که اثباتش آسونه.
لم2:در چهارضلعی ABCD نقاط M,N,P,Q به ترتیب بر اضلاع AB,BC,CD,DA قرار دارند به گونه ای که:

حال اگر محل برخورد MP و NQ را T بنامیم خواهیم داشت:

که اثباتش آسونه.
در مسئله اصلی محی برخورد AD,BC را E و محل برخورد AB,CD را F بنامید.همچنین حل برخورد YT و ZX رو P بنامید. توجه کنید لم بالا در حالتی که نقاط بر امتداد اضلاع قرار داشته باشند نیز برقرار است. پس داریم:

نقطه میشل چهارضلعی را M بنامید. همواره برای این نقطه داریم:
. با توجه به اینکه:
داریم:

پس MTYE و به طور مشابه MXZF محاطی هستند.

پس بنا بر لم 1، MP بر دایره محیطی MTYE مماس است. به طور مشابه بر دایره محیطی MXZF مماس است.پس بر روی محور اصلی این دو دایره قرار دارد و در نتیجه:


و حکم ثابت می شود.
سوال خفنی بود.
 

olampiad2

New Member
ارسال ها
28
لایک ها
0
امتیاز
0
پاسخ : ماراتن هندسه

لم 1:در مثلث ABC، مماس وارد بر دایره محیطی در نقطه A، امتداد ضلع BC را در D قطع می کند.حال داریم:

که اثباتش آسونه.
لم2:در چهارضلعی ABCD نقاط M,N,P,Q به ترتیب بر اضلاع AB,BC,CD,DA قرار دارند به گونه ای که:

حال اگر محل برخورد MP و NQ را T بنامیم خواهیم داشت:

که اثباتش آسونه.
در مسئله اصلی محی برخورد AD,BC را E و محل برخورد AB,CD را F بنامید.همچنین حل برخورد YT و ZX رو P بنامید. توجه کنید لم بالا در حالتی که نقاط بر امتداد اضلاع قرار داشته باشند نیز برقرار است. پس داریم:

نقطه میشل چهارضلعی را M بنامید. همواره برای این نقطه داریم:
. با توجه به اینکه:
داریم:

پس MTYE و به طور مشابه MXZF محاطی هستند.

پس بنا بر لم 1، MP بر دایره محیطی MTYE مماس است. به طور مشابه بر دایره محیطی MXZF مماس است.پس بر روی محور اصلی این دو دایره قرار دارد و در نتیجه:


و حکم ثابت می شود.
سوال خفنی بود.
لطفا یکی سوال بعدو بذاره

---- دو نوشته به هم متصل شده است ----

لم 1:در مثلث ABC، مماس وارد بر دایره محیطی در نقطه A، امتداد ضلع BC را در D قطع می کند.حال داریم:

که اثباتش آسونه.
لم2:در چهارضلعی ABCD نقاط M,N,P,Q به ترتیب بر اضلاع AB,BC,CD,DA قرار دارند به گونه ای که:

حال اگر محل برخورد MP و NQ را T بنامیم خواهیم داشت:

که اثباتش آسونه.
در مسئله اصلی محی برخورد AD,BC را E و محل برخورد AB,CD را F بنامید.همچنین حل برخورد YT و ZX رو P بنامید. توجه کنید لم بالا در حالتی که نقاط بر امتداد اضلاع قرار داشته باشند نیز برقرار است. پس داریم:

نقطه میشل چهارضلعی را M بنامید. همواره برای این نقطه داریم:
. با توجه به اینکه:
داریم:

پس MTYE و به طور مشابه MXZF محاطی هستند.

پس بنا بر لم 1، MP بر دایره محیطی MTYE مماس است. به طور مشابه بر دایره محیطی MXZF مماس است.پس بر روی محور اصلی این دو دایره قرار دارد و در نتیجه:


و حکم ثابت می شود.
سوال خفنی بود.
لطفا یکی سوال بعدو بذاره
 

Amir m

New Member
ارسال ها
1
لایک ها
0
امتیاز
0
پاسخ : ماراتن هندسه

در مثلث abc , نقاط m و n روی اضلاع ab و ac طوری انتخاب میکنیم که محل تقاطع آنها روی ah ( ارتفاع وارده بر ضلع bc از راس a ) باشد. ثابت کنید ارتفاع ah نیمساز زاویه mhn میباشد.
 

olampiad2

New Member
ارسال ها
28
لایک ها
0
امتیاز
0
پاسخ : ماراتن هندسه

در مثلث abc , نقاط m و n روی اضلاع ab و ac طوری انتخاب میکنیم که محل تقاطع آنها روی ah ( ارتفاع وارده بر ضلع bc از راس a ) باشد. ثابت کنید ارتفاع ah نیمساز زاویه mhn میباشد.
سوالی که در این زمینه مطرح میشه اینه که محل تقاطع bn و cm رو ah است.اثبات : خط موازی bcرا می کشیم تا hn و hm را به ترتیب در xوyقطع کند. واضحه که مثلث axm با bmh ومثلث ayn با nhc متشابه است پس اگر از دو رابطه تشابه و قضیه سوا استفاده کنیم نتیجه میشه که ax=ay پس مثلث hxyمتساوی السلقین و ah نمیسازه.

---- دو نوشته به هم متصل شده است ----

سوال بعد: مثلث حاده الزاویه ABCمفروض است.پای ارتفاعات وارد از A,B,Cرا به ترتیب D,E,Fمی نامیم و پای عمود های وارد ازB,Cبر خطوط FD,DEرا Y,Z می نامیم.سپس قرینه Fنسبت به Eرا Xوقرینه E نسبت به F را T. اگر 3EF= FD+DEثابت کنیدزوایای BZXوCYTبرابرند.
 
آخرین ویرایش توسط مدیر

Dadgarnia

New Member
ارسال ها
1,350
لایک ها
1,127
امتیاز
0
پاسخ : ماراتن هندسه

نقاط A,B,C مراکز دوایر محاطی خارجی مثلث DEF هستند پس Y,Z هم نقاط تماس دایره محاطی خارجی نظیر رئوس E,F با اضلاع FD,ED هستند. به راحتی و با استفاده از شرط سوال داریم FY=EF=EZ پس توی مثلث های EYT و FZX میانه نصف وتره که نتیجه میده EYT=FZX=90 پس کافیه نشون بدیم EYC=FZB. چون مثلث EFY متساوی الساقینه و FC نیمساز راس F خط FC بر EY عموده در نتیجه EY موازیه با AB و به طور مشابه FZ موازیه با AC پس ZFB=BAC=YEC. حالا به راحتی و با استفاده از قضیه سینوس ها می تونیم بدست بیاریم YE/ZF=cos C/cos B=CE/BF پس دو مثلث YEC و ZFB با هم متشابه اند که حکمو نتیجه میده.
سوال بعد:
با همون فرض های سوال قبل نشون بدین وسط AD مرکز ارتفاعی مثلث ABC هست.
 

assassin1

New Member
ارسال ها
80
لایک ها
18
امتیاز
0
پاسخ : ماراتن هندسه

اثبات:مثلث DEFرا درنظر می گیریم پای ارتفاع وارد ازDرا Tمیگیریم داریم 2S_def =2rP=DT×EF که rمرکز دایره محاطی داخلی است پس با فرض مسئله داریم TD=4r حال با کمی دقت و استفاده از قضیه تالس از نتیجه آخر نتیجه میشه که: AI=II_aکه همون حکمه.
 
آخرین ویرایش توسط مدیر

assassin1

New Member
ارسال ها
80
لایک ها
18
امتیاز
0
پاسخ : ماراتن هندسه

نقطه دلخواه oدر مثلثabc مفروض است. N,mپای عمود هایی هستند که از oبر نیمساز های داخلی و خارجی راسaرسم می شوند . به همین ترتیب q,p برای b وt,rبرای c تعریف می شوند.اگر محل تقاطع mn,pqرا sبنامیم. ثابت کنید s,R,Tهمخطند.
 
آخرین ویرایش توسط مدیر
ارسال ها
288
لایک ها
154
امتیاز
43
پاسخ : ماراتن هندسه

نقطه دلخواه oدر مثلثabc مفروض است. N,mپای عمود هایی هستند که از oبر نیمساز های داخلی و خارجی راسaرسم می شوند . به همین ترتیب q,p برای b وt,rبرای c تعریف می شوند.اگر محل تقاطع mn,pqرا sبنامیم. ثابت کنید s,R,Tهمخطند.
تو كتاب هندسه مسطحه احمد پور تو قسمت سوا اگه اشتباه نكنم مثال اوله

---- دو نوشته به هم متصل شده است ----

نقطه دلخواه oدر مثلثabc مفروض است. N,mپای عمود هایی هستند که از oبر نیمساز های داخلی و خارجی راسaرسم می شوند . به همین ترتیب q,p برای b وt,rبرای c تعریف می شوند.اگر محل تقاطع mn,pqرا sبنامیم. ثابت کنید s,R,Tهمخطند.
تو كتاب هندسه مسطحه احمد پور تو قسمت سوا اگه اشتباه نكنم مثال اوله
 

assassin1

New Member
ارسال ها
80
لایک ها
18
امتیاز
0
پاسخ : ماراتن هندسه

مثلث abcمفروض است.برروی اضلاع acوbcوabمثلث هایی متساوی الساقین می سازیم به طوریکه اضلاع ذکر شده قاعده ها باشند و به ترتیب راس هایdوeوfبا زوایای برایر بدست آیند.ثابت کنید aeوbdوcfهمرسند.ثابت کنید نیمساز های رئوس dوeوfهمرسند.نشان دهید اگر اوساط acوbcوabرا lوhوgبنامیم انگاه dlوehوfgهمرسند.همچنین اگر tو'tو"tپای عمود های وارد ازdوeوfبراضلاع مثلث abcباشند انگاه dtوe'tوf"tهمرس خواهند شد.
 
آخرین ویرایش توسط مدیر
بالا